Classroom Logo

  • Teacher Opportunities
  • AP U.S. Government Key Terms
  • Bureaucracy & Regulation
  • Campaigns & Elections
  • Civil Rights & Civil Liberties
  • Comparative Government
  • Constitutional Foundation
  • Criminal Law & Justice
  • Economics & Financial Literacy
  • English & Literature
  • Environmental Policy & Land Use
  • Executive Branch
  • Federalism and State Issues
  • Foreign Policy
  • Gun Rights & Firearm Legislation
  • Immigration
  • Interest Groups & Lobbying
  • Judicial Branch
  • Legislative Branch
  • Political Parties
  • Science & Technology
  • Social Services
  • State History
  • Supreme Court Cases
  • U.S. History
  • World History

Log-in to bookmark & organize content - it's free!

  • Bell Ringers
  • Lesson Plans
  • Featured Resources

sunshinecavalluzzi Thumbnail

Lesson Plan: AP Government: Argumentative Essay Practice

Red Arrow

The Federalist Papers

Boston College professor Mary Sarah Bilder gives a brief overview backgrounding the Federalist Papers

Description

This is intended as an end-of-course review activity for practice with the argumentative essay format included on the AP United States Government and Politics exam since the 2018 redesign. Eleven practice prompts are provided, reflecting content from Units 1-3.

ARGUMENTATIVE ESSAY PROMPT ANALYSIS

  • Review the provided Argumentative Essay Prompts in either an individual or jigsaw format.
  • Write a thesis statement for your selected prompt(s) and identify the selection you would make from the provided list and the second piece of evidence you would choose.
  • If there are prompts for which you struggle to develop a thesis, or items on the bulleted lists with which you are not conversant, use the hyperlinked C-SPAN Classroom resources to extend your understanding of the required founding documents and SCOTUS cases that you found challenging.

ARGUMENTATIVE ESSAY

  • Chose one or more of the provided Argumentative Essay Prompts , as assigned, and use the planning and exploration you did above to write a full essay in response to your designated prompt(s) in 25 or fewer minutes , since that's the time limit you'll face on the AP Exam!
  • Exchange essays with a classmate and evaluate each others' work.
  • 1st Amendment
  • Branches Of Government
  • Constitution
  • House Of Representatives
  • Separation Of Powers
  • Supreme Court

AP US Gov FRQ: Argument Essay Review (2020)

6 min read • june 18, 2024

Fatima Raja

Fatima Raja

So, you’re reading this article and wondering how to approach the APGOPO and CompGov Argument Essay. First of all, no, this is not like a dinner table argument over politics that happens in every movie at Thanksgiving. Unlike one of those discussions, you have to actually use facts to get your point across (😂), and the reader is more concerned about your line of reasoning than about their own political opinions 👀.  

Let’s break down exactly what you should expect, so you can craft a solid argument:

In total, you have an hour and 40 minutes to finish the entire Free-Response Question section if you're in APGOPO. If you're in APCompGov, you have an hour and 30 minutes It’s important to use your time effectively because the FRQ section is worth half of your score. 

Because of that, you should spend around 25 minutes, give or take a few, on the Argument Free-Response Question. (NOTE: FOR THE 2019-2020 TEST, YOU WILL HAVE 25 MINUTES TO WRITE AND 5 MINUTES TO UPLOAD YOUR RESPONSE.)

ap gov argument essay example 2021

This is the nightmare you’re  not gonna have before this AP exam. 

Image courtesy of Freepik.

Obviously, you want to be able to conserve time and learn how to do these FRQs as effectively and quickly as possible. Here’s the secret to doing that: practice! You have to apply the concepts you’re learning to actual questions, so you understand how to break them down when you’re under pressure.

That way, you can get these skills down to muscle-memory and not be too stressed when you get to test day!

Structure (What does the FRQ look like?)

You’re gonna get a prompt that you have to write about (duh), but here’s the thing the point is not to explain or restate the prompt. The point is to  develop an argument based on it .

Don’t just write about the situation that the prompt sets up. You need to explain why we should or should not do whatever the example is. This is an argument essay, so you need to  argue a position. It doesn’t have to be the “right” position. It just needs to be logical and supported with evidence.

This is what you’ll be given to do just that:

A prompt.  You have to explicitly agree or disagree with it when you state your thesis!

A few foundational documents.  You’re required to use at least one example that is listed, so make sure you know them.

  • Note for CompGov: Sorry, you don’t get these. Just make sure you include a specific piece of evidence. But, you also need a few other things to actually get all the points available:

An   additional piece of evidence. This can be a different foundational document than the one you initially used or any specific concept from APGOPO or CompGov.

Analysis. You have to explain why your evidence justifies your line of reasoning (aka your thesis).

An alternate perspective.  Not everyone will agree with the position you take. That’s the beauty of democracy. To show you understand that, you have to refute your point or provide some concession to another POV (NOTE: THIS IS NOT NEEDED FOR THE 2019-2020 TEST.) Now that you know what to expect, let’s figure out how to tackle the Argument FRQ!

How to Tackle the Argument Essay

Here’s what you need to do to tackle this FRQ thoroughly:

Look at the prompt and start thinking of a thesis.  You may have a personal opinion right after looking at the prompt, or you may not. Either way, as you start to look at the provided documents, start brainstorming how you want to write your essay. It’s okay if this changes when you see what evidence is provided or what evidence you come up with on your own. It’s just good to have a jumping board.

Analyze the documents!  This means you need to look at each of the foundational documents (again, CompGov, you don’t get any) and figure out how they fit into the context of the prompt. Does the evidence in question agree or disagree with the situation the prompt presents? How can  you  use it to support or refute your argument?

Create an outline.  This is a good way to figure out exactly what you’re going to say, and you know what evidence you’re using. This will help you have a clear, well-thought out essay. Your outline shouldn’t be incredibly detailed, though! You still have to transfer everything to your writing booklet before you run out of time.

State your thesis.  This is critical to ensuring you get full points. If the prompt asks whether or not America should switch to being a direct democracy, don’t just give a wishy-washy list of pros and cons.  Make your answer explicit : “Yes, America should transition to being a direct democracy because x and y.” or “No, America should not become a direct democracy because of p and q.” Don’t turn your essay into a treasure hunt for your argument, just state it plainly.

ap gov argument essay example 2021

POV: you’re reading this article, taking notes, and getting ready to win this FRQ game.

Image courtesy of Pixabay

Provide justification!  You already know that you need  two pieces of evidence , but that’s not all. You can’t just say something like “The Articles of Confederation show that we should have a strong federal government.” It should be more like, “The problematic Articles of Confederation only further exemplify why we need to have a strong federal government, as expanded federal power is necessary to prevent the dissolution of the Union by dangerous uprisings, such as the Whiskey Rebellion.” You need to explain why each piece of evidence strengthens your argument. Don’t just toss in a vague reference and call it a day.

Consider an alternate perspective.  This is critical to ensuring you get full points. Showing that you understand that your argument isn’t the only way to approach a situation shows you understand that every approach has its pros and cons. So, refute your argument or explain a situation in which it may not apply. Pro tip: showing why the example you used to weaken your argument is wrong only strengthens it! (NOTE: THIS IS NOT NEEDED FOR THE 2019-2020 TEST.)

Some Final Tips!

Practice! Practice! Oh, and did I say that you need to practice?  Getting familiar with the structure and time constraints you’ll be under when writing the argument essay will allow you to actually get comfortable with it. You’ll understand how to apply the strategies I just talked about and discover some of your own!

Breathe!  Don’t freak out. You may be feeling the pressure, but you’ve been prepping for this all year (or all semester). You put in the work, and you’ll be fine! Keeping a cool head will help you get the best score you can.

ap gov argument essay example 2021

Practice Prompts!

Here are some prompts to get you started:

The right to free speech for all citizens is protected in America. Though the Supreme Court has limited journalistic expression in some cases and individual states have worked to restrict the right of citizens to assemble, free speech remains a defining pillar of American society.

Develop an argument about whether restrictions on free speech and assembly ultimately help or hinder democracy.

Use at least one piece of evidence from one of the following foundational documents:

  • Bill of Rights
  • Letter from Birmingham Jail
  • Federalist 10

Since 200, Russian President Vladimir Putin has ensured that he wins Russian presidential elections. The presidential elections are rigged for Putin to win and to demonstrate his hold on Russian society. Russian opposition parties have gained some ground in recent years, seemingly in spite of Kremlin interference.

Develop an argument about whether elections have strengthened or weakened democracy in Russia.

Watch:  Argumentative Essay Writing Workshop

Fiveable

Stay Connected

© 2024 Fiveable Inc. All rights reserved.

AP® and SAT® are trademarks registered by the College Board, which is not affiliated with, and does not endorse this website.

AP U.S. Government and Politics Exam Tips

The following strategies for answering the free-response questions will help you on exam day.

  • Answering essay questions generally requires a good deal of training and practice. Students too often begin to write immediately, which can create a string of disconnected, poorly planned thoughts. You should approach questions methodically and plan your answers before putting pencil to paper.
  • Carefully analyze the question, thinking through what is being asked, and identify the elements that must be addressed in the response. Each AP Exam asks different types of questions about each subject. For example, some questions may require you to consider the similarities between people or events, and then to think of the ways they are different. Others may ask you to develop an argument with examples in support of or opposing a particular movement or policy. Be sure to carefully craft your answer in response to what is actually being asked in the question prompt.
  • After you have determined what is involved in answering the question, consider what evidence you can incorporate into your response. Review the evidence you learned during the year that relates to the question and then decide how it fits into the analysis. Does it demonstrate a similarity or a difference? Does it argue for or against a generalization that is being addressed?
  • Whenever you offer evidence to illustrate contrast or similarity, clearly state your intent. Then, with additional information or analysis, elaborate on the ways in which these pieces of evidence are similar or different. If there is evidence that refutes a statement, explain why it argues against the statement. Your answer should reflect an understanding of the subtleties of the questions.
  • Begin writing only after you have thought through the evidence you plan to use and have determined what your thesis statement will be. Once you have done this, you will be in a position to answer the question analytically instead of in a rambling narrative.
  • Learn how to present your thesis statement: make your overarching statement or argument and then position your supporting evidence so that it is obviously directed to answering the question, as opposed to being a string of abstract generalizations. State your points as clearly as possible and explicitly connect them to the larger thesis. Do not leave it to the reader to infer what is meant or how something illustrates a point.
  • If you have done the analytical work required prior to writing, you should be able to demonstrate an understanding of the complexity of the question. You should state your thesis, introduce the elements that support the thesis, and demonstrate the logic that led you to link the elements in support of the thesis. By applying these ideas you will construct an excellent essay.
  • While essay writing in general is a valuable exercise, you may wish to work specifically on free-response questions from previous AP Exams. This will allow you to compare your own responses with those that have already been scored and evaluated. Free-response questions are available through the AP Program in numerous formats. Fnd sample essays for U.S. Government and Politics on the Past Exam Questions page at AP Central.

Pay close attention to the task verbs in the question. Each one directs you to complete a specific type of response. Here are the task verbs you’ll see on the exam:

  • Compare : Provide a description or explanation of similarities and/or differences.
  • Define : Provide a specific meaning for a word or concept.
  • Describe : Provide the relevant characteristics of a specified topic.
  • Develop an argument : Articulate a claim and support it with evidence.
  • Draw a conclusion : Use available information to formulate an accurate statement that demonstrates understanding based on evidence.
  • Explain : Provide information about how or why a relationship, process, pattern, position, situation, or outcome occurs, using evidence and/or reasoning. Explain “how” typically requires analyzing the relationship, process, pattern, position, situation, or outcome; whereas explain “why” typically requires analysis of motivations or reasons for the relationship, process, pattern, position, situation, or outcome.
  • Identify : Indicate or provide information about a specified topic, without elaboration or explanation.

ap gov argument essay example 2021

Sample Prompts for the Argument Essay FRQ- AP government

ap gov argument essay example 2021

Below are 16 topics, each of which includes:

  • A sample essential question which introduces the prompt
  • A draft prompt including three founding documents that could help shape the students’ arguments.

Each prompt is crafted to encourage deep analysis and aligns with key AP Government concepts, ensuring your students are well-prepared for exam success.

AP Government Argument Essay Samples

  • NEW ! Media censorship: Should the government play an active role in the censorship of social media?  
  • Independent judiciary:   Is an independent judiciary a threat to or a savior for democracy?  
  • Congressional roles:   Does the delegate or trustee model of Congressional representation best serve the needs of the people as the Framers intended?  
  • Federalism in the Age of Coronavirus:  Should the federal government or the states be most responsible for responding to the Coronavirus outbreak?  
  • Political Parties:  Do political parties hinder or promote democracy?  
  • Congressional oversight :  Is congressional oversight healthy or unhealthy for our system of government?
  • Interest groups: Do interest groups hinder or promote democracy? 
  • Civil Rights:  Should the federal government have power over states in the shaping of civil rights policies?
  • Citizen participation: Does citizen participation really matter? 
  • Photo IDs and federalism:   Do states have the authority to pass photo identification laws which restrict people’s ability to vote?  
  • Presidential power:   Do executive orders give the president too much power?      
  • Gridlock:   Is gridlock healthy or unhealthy for our system of government?
  • Term limits:   Do congressional term limits violate or honor popular sovereignty?
  • Primaries and caucuses: Is the presidential nominating process democratic? 
  • Social Media :  Is social media a healthy way for citizens to participate in our political system?  
  • Electoral College:   Should the electoral college be abolished?
  • Representative versus direct democracy: Which is a better vehicle to serve citizen needs– a representative or direct democracy?  

Enhance Your Classroom Experience! You understand the challenges of keeping students engaged and preparing them for the AP exam. Our carefully curated essay prompts are designed to align with AP standards, fostering critical thinking and discussion in your classroom. Get ready to inspire your students with materials that cater to the dynamic world of government and politics.

For more resources for AP government, visit HERE

Other posts you might like…, you may also like.

ap gov argument essay example 2021

The Secret to Mastering Critical Thinking in an AI Era

ap gov argument essay example 2021

If You’re Trying to Figure Out When Robots Will Replace Teachers, You Came to the Right Place

ap gov argument essay example 2021

AI in Education: A Game Changer or a Double-Edged Sword?

ap gov argument essay example 2021

The Human Element in AI-Driven Classrooms

Copyright © 2023 Teach Different. All rights reserved.

Connect with:

ap gov argument essay example 2021

Login with your site account

ap gov argument essay example 2021

Remember Me

Not a member yet? Register now

Register a new account

I accept the Terms of Service

Are you a member? Login now

ap gov argument essay example 2021

Don't miss out on the conversation! Subscribe to our blog

Email Address

Subscribe Now

logo-type-white

AP® US Government

How to answer ap® us government free response questions.

  • The Albert Team
  • Last Updated On: March 1, 2022

how to answer AP® US Government free response questions

Mastering the free response section can make or break any student’s AP® US Government and Politics score. If you’re looking for the best tips and tricks for answering AP® US GoPo free response questions, you’ve come to the right place. 

In this article, we’ll review tips for writing top-mark AP® US Government and Politics FRQs, mistakes that students make one too many times on past AP® GoPo exams, and how to use past AP® free response questions to start practicing for your upcoming exam. 

Keep reading to get everything you need to know when it comes to making the most of your AP® US Government and Politics exam review. 

What We Review

5 Steps on How to Write Effective AP® US Government and Politics Free Responses

Here, we’ll review a five-step strategy for you to start writing AP® US GoPo free response answers that will score you maximum possible points. 

1. Commit to learning what gets you points on the AP® US Government and Politics exam by reviewing past rubrics and scoring guidelines.

A common mistake students make when it comes to preparing for their AP® GoPo exam is failing to understand how they’re being graded. The first step to solving this is going to the College Board’s AP® Central website and navigating to the past released exams for US Government. 

Here is the link for AP® US Government and Politics past released exams

Open up the scoring guidelines PDF. These guidelines outline how points were distributed on that particular year’s exams. 

Here’s a screenshot from the first question of the 2019 released exam: 

AP® US Government frq

Source: College Board

From this, you can see that this short answer question in 2019 was worth three points, with one point being allocated to each part. There are certain directive words to keep an eye on when reviewing AP® US Government and Politics free response questions, but we’ll get into that later.

For now, just make sure you review at least two years worth of released exam scoring guidelines so you begin to understand how questions and parts of questions are weighted. 

2. Underline or circle every bolded and capitalized word in the question prompt.

Alright, so now that we know how points are generally distributed, we need to build the habit of having a system for ensuring we actually answer the question asked by the College Board when we start our AP® US Government and Politics free response section.

ap us gopo frq example

AP® US Government and Politics isn’t as “nice” as AP® Biology free response questions in that they don’t always bold key directive words for you to know what is being asked.

That being said, it’s not hard to circle or underline for yourself the key thing you are being asked to answer. 

There are three “key phrases” to commit to memory when it comes to AP® US Government and Politics free response questions: 

That’s it. If you look at the last few years worth of released exam questions, these are the most commonly used directive words for the short answer question section of the AP® US Government and Politics free response section. 

If you aren’t sure what the three of these words are asking you for, keep reading.

When the exam asks you to describe something, you need to tell them about what they’re asking. This doesn’t mean you need to explain the “why” — it just means you need to talk about what the topic is and the characteristics of the topic being asked.

When you’re asked to explain something, this is where you need to show the “why”. You need to be able to give 3-5 sentences with an example in most cases to earn credit for these questions.

Finally, when asked to identify something, you need to simply indicate that you know what the topic is related to — no need for explanation or elaboration as you might when asked to describe or explain. 

One of our best test taking tips we can give you is to make a tick mark or star next to the words you have circled or underlined after you’ve answered it in your free response. This serves as a visual checklist for you to make sure you answered all parts of the question. 

Trust us! It’s easy to forget to answer one small part of an FRQ, and that can make all the difference in your free response score. 

Aside from the three directive words above, other commonly used ones for AP® GoPo include:

  • Define : Similar to identify — show that you know what the topic is but there is no need to elaborate further than what’s asked.
  • Compare : Provide a description of the similarities and/or differences of the topics presented.
  • Develop an Argument : State a claim and support it with evidence.
  • Draw a Conclusion : Make an accurate statement from what has been presented. 

3. Plan your response BEFORE beginning to write your response. 

planning your AP® US Government frq out

One of the most commonly cited mistakes students make on the AP® GoPo free response section is not actually answering the question in a thoughtful way. 

The College Board uses the free response section to test your ability to connect the dots with what you’ve learned in class. You need to demonstrate skills such as considering evidence to incorporate and how that fits into your analysis. 

This means plan out your response before you begin writing! 

Take a second before putting your pen down to start writing to think through how you’ll answer the “why” based questions. 

Think deeply about what the question is actually asking you — sometimes students answer questions without actually…answering the question.

Readers often express that student misconceptions come from having a poorly planned response or simply restating the question without adding any direct response to the question they were asked. 

4. Remember that AP® US Government and Politics free responses are not like other subjects — treat them differently than you may in AP® English Language. 

ap us gopo frq

When it comes to the short answer questions in AP® GoPo, you do not need to write an essay to score max points. There is no need for an introduction, thesis, or conclusion on these questions.

When it comes to the argumentative essay, it’s not necessarily a cookie-cutter five paragraph essay either. 

The argumentative essay’s scoring depends on each proceeding section building on the prior. On every question 4, the College Board states exactly what you need to score maximum possible points. 

You need: 

  • To articulate a defensible claim or thesis that responds to the prompt and establishes a line of reasoning.
  • Typically one will be from a foundational document while the other will be any other foundational document you learned in class
  • Use reasoning to explain why your evidence supports your claim or thesis
  • Respond to an opposing or alternative perspective using refutation, concession, or rebuttal.

What this means is that as long as you cover all the points outlined above clearly, you can score a perfect score on the argumentative essay! 

When it comes to preparing for the argumentative essay, one of the best things you can do is make sure you are fully comfortable with all 9 foundational documents and 15 Supreme Court cases. 

The required foundational documents to know are: 

  • The Declaration of Independence
  • The Articles of Confederation
  • The Constitution of the United States
  • Federalist No. 10
  • Brutus No. 1
  • Federalist No. 51
  • Federalist No. 70
  • Federalist No. 78
  • Letters from a Birmingham Jail

Kelsey Falkowski has a nice 15-minute review video of these foundational documents here .

The required Supreme Court cases are:

  • Marbury v. Madison (1803)
  • McCulloch v. Maryland (1819)
  • Schenck v. United States (1919)
  • Brown v. Board of Education of Topeka (1954)
  • Baker v. Carr (1961)
  • Engel v. Vitale (1962)
  • Gideon v. Wainwright (1963)
  • Tinker v. Des Moines Independent Community School District (1969)
  • New York Times Company v. United States (1971)
  • Wisconsin v. Yoder (1972)
  • Roe v. Wade (1973)
  • Shaw v. Reno (1993)
  • United States v. Lopez (1995)
  • McDonald v. Chicago (2010)
  • Citizens United v. Federal Election Commission (2010)

Adam Norris has a great 11-minute review video on these fifteen cases here .

Typically when it comes to the final component, we like using rebuttals more than concessions or refutations. The reason why is because when you make a concession or refutation on a claim you made earlier in your essay, it can potentially come across as a weakening of your thesis if you are not able to position it properly. 

The last thing to remember here is to make sure you “close the loop”. This is a test taking strategy the College Board promotes across multiple disciplines and with good reason — it challenges a student to demonstrate they can form a coherent argument. Closing the loop in AP® US Government can mean using words like “because” or “therefore” to help bridge two concepts together and solve for the “why” this matters. 

5. Practice, practice, and did we say practice?

When you reduce AP® free response sections down to their core, regardless of subject domain, mastering them comes down to two things: knowing how you’re going to be graded, and learning how to craft responses that fit those rubrics. 

Sometimes students do the first part well but fail to practice enough at doing the second part, and vice versa. 

When you first start out, we recommend trying a set of past released questions and then having a friend grade your responses with the scoring guidelines. See how you would do without any intentional prep. Then, learn from your mistakes, log your mistakes in a study journal, and begin intentionally tackling other years one by one.

After a few times of doing this, you’ll begin building your intuition to craft a perfect-score response for your AP® US Government and Politics free responses.

25 AP® US Government and Politics FRQ Tips to Scoring a 4 or 5

Now that we’ve gone over the 5-step process to writing good AP® GoPo free responses, we can shift gears to tackle some test taking tips and tricks to maximizing your FRQ scores. 

We recommend you review these several weeks, and then days before your exam to keep them top of mind. 

  • Know which SAQ you’re weakest at. There are always three core question types: concept application, quantitative analysis, and SCOTUS comparison. If you’re weak at one, make sure you’ve reviewed all the past released exams for that particular SAQ. 
  • Make sure you review how issues or ideology can drive partnership on specific issues. 
  • Focus on applying the political concepts and processes you learned in class to scenarios in context. This is one of the most common mistakes for the SAQs. 
  • One of the easiest ways to bridge two concepts is to use words like “because” or “therefore” and then proceed to answer the “why this matters”. 
  • Focus on what is right instead of what is wrong in your response. These free response questions are often graded based on what’s right more so than what’s wrong (which is different from another subject like AP® Biology).
  • If you’re not 100% sure about a supporting statement, add a second supporting statement on the topic as backup. 
  • If you’re offering specificity, make sure to be explicit on what the intent of you introducing that in your response is. For example, if something is being presented to rebuttal something else, explain why or how it does so. 
  • When it comes to data analysis, you need to make sure you are comfortable interpreting data and applying data to demonstrate how it interacts with the political process. 
  • In the past, students have not been able to analyze and apply data to course content — they make mistakes in connecting how policy relates to respective parties in the political process. 
  • When practicing data analysis, it’s important to look at a variety of different types of graphs and focus on identifying the similarities and differences within a set of complex data. 
  • Data analysis is not just reading graphs, but also reading charts and tables. Don’t just think because you got one question reading a graph correctly that you’re good to go for your quantitative analysis SAQ. 
  • One of the easiest ways to bolster your data analysis skills is by reviewing sources such as the Gallup National Polls or Pew Research findings. 
  • When it comes to SCOTUS comparison, students often fail to effectively compare the two cases — they do a fine job of recalling the required case, but struggle to connect the required case to the non-required case. 
  • Remember that the SCOTUS comparison SAQ is typically not going to ask you to discuss the rulings of the required case, but rather the facts of the cases and how it applies to the non-required case. 
  • Keep an eye out for when you’re asked for the clause from an amendment or the Constitution. This means there will only be one right answer. 
  • Know the difference between reasoning of a case, the decision, opinions of the case, as well as the cold hard facts. Make flashcards or use Quizlet to help here. 
  • When you’re asked to compare facts, it means you need to review the facts of both cases, not just one. Even if the facts for the non-required case are included in the prompt, you need to include it in your response for points. 
  • When it comes to the argumentative essay, students typically fail to explain why the evidence they bring up supports their thesis.
  • The second area students struggle is in responding to an alternative perspective (refutation, concession, or rebuttal).
  • X is your counterargument or counterpoint
  • ABC are your strongest supporting points for your argument.
  • And Y is your argument. 
  • Know your foundational documents cold. Sometimes students mix up these documents. There are four different Federalists to know!
  • When looking to get the reasoning point, make sure to explain why the evidence you’re procuring supports your thesis. Don’t just restate your thesis or state the evidence without connecting the two. 
  • When seeking your perspective point (for refutation, concession, or rebuttal), make sure to state the alternative point of view, but also respond to it. Both of these parts are needed.
  • Work with a friend through at least three years of AP® GoPo FRQs. Then swap with each other and go through the scoring guidelines together to get consistent exposure to the rubrics. 
  • By your last two weeks before the exam, you should have clearly identified your 3-5 biggest weaknesses when it comes to FRQs. Devote at least 70% of your time to these areas and the remaining time on general review. 

Wrapping Things Up: How to Write AP® US Government and Politics FRQs

Wow! We’ve gone over a ton of things in this AP® US Government and Politics review guide. At this point, you should have everything you need to get started in preparing for your GoPo FRQs. 

To summarize, here are a few things to remember: 

  • Great AP® US Government and Politics free response scores are only made when you know how you’re being graded. Learn the rubrics. 
  • Have a consistent system for responding to each question. We recommend circling or underlining what you’re being asked, and then adding a tick or star next to the word in the prompt when you’ve answered it.
  • Know the facts of your foundational documents and required Supreme Court cases cold. Students have missed points in the past by mixing up one with another. 
  • Practice working with multiple types of data for the quantitative analysis SAQ: this means reviewing charts, graphs, and tables. Focus on being able to interpret the data presented to political concepts or processes. 
  • Review commonly tested AP® US Government and Politics topics. Review the curriculum and exam description to see the percentage breakdown of different units. Unit 2 on Interactions Among Branches of Government is a very important one to know as it makes up 25-36% of the exam. 
  • Make sure your thesis includes a clear line of reasoning. Remember the model: Although X, ABC, therefore Y.
  • Always “close the loop”. Use words such as “because” or “therefore” to bridge two concepts together and solve for the “why” this matters. 

We hope you’ve found this exhaustive guide helpful for your AP® US Government and Politics exam review. 

If you’re looking for more free response questions or multiple choice questions, check out our website! Albert has hundreds of original standards-aligned practice questions for you with detailed explanations to help you learn by doing.

If you found this post helpful, you may also like our AP® US Government tips here or our AP® US GoPo score calculator here .

We also have an AP® US Government review guide here .

Interested in a school license?​

2 thoughts on “how to answer ap® us government free response questions”.

On the list of required supreme court cases, you guys listed Schneck v United States with the wrong year.

Great catch, Johnny! We’ve updated the list to include the correct date (1919).

Comments are closed.

Popular Posts

AP® Physics I score calculator

AP® Score Calculators

Simulate how different MCQ and FRQ scores translate into AP® scores

ap gov argument essay example 2021

AP® Review Guides

The ultimate review guides for AP® subjects to help you plan and structure your prep.

ap gov argument essay example 2021

Core Subject Review Guides

Review the most important topics in Physics and Algebra 1 .

ap gov argument essay example 2021

SAT® Score Calculator

See how scores on each section impacts your overall SAT® score

ap gov argument essay example 2021

ACT® Score Calculator

See how scores on each section impacts your overall ACT® score

ap gov argument essay example 2021

Grammar Review Hub

Comprehensive review of grammar skills

ap gov argument essay example 2021

AP® Posters

Download updated posters summarizing the main topics and structure for each AP® exam.

PrepScholar

Choose Your Test

Sat / act prep online guides and tips, the complete guide to the ap comparative government and politics exam.

author image

Advanced Placement (AP)

feature-government-1-cc0

The AP Comparative Government and Politics exam tests your knowledge of how the political systems in different countries are similar and different. The exam requires endurance, strong critical thinking, and top-notch writing skills…which means you’ll need to be extra prepared!

If you’re looking for an AP Comparative Government study guide to carry you through all of your AP prep, look no further than this article! We’ll walk you through: 

  • The structure and format of the AP Government — Comparative exam
  • The core themes and skills the exam tests you on
  • The types of questions that show up on the exam and how to answer them (with sample responses from real AP students!)
  • How the AP Comparative Government exam is scored, including official scoring rubrics
  • Four essential tips for preparing for the AP Comparative Government exam

Are you ready? Let’s dive in!

feature-government-2-cc0

Understanding how major world governments work will be key to doing well on this exam!

Exam Overview: How Is the AP Government — Comparative Exam Structured?

First things first: you may see this exam referred to as both the AP Government — Comparative exam  or t he AP Comparative Government exam. Don't worry, though...both of these names refer to the  same  test! 

Now that we've cleared that up, let's look at the structure of the test itself. The AP Comparative Government and Politics exam tests your knowledge of basic political concepts and your ability to compare political systems and processes in different countries. 

This AP exam is on the shorter side, lasting for a total of two hours and 30 minutes . You’ll be required to answer 55 multiple-choice questions and four free-response questions during the exam. 

The AP Comparative Government exam is broken down into two sections . Section I of the exam consists of 55 multiple-choice questions and lasts for one hour. The first section of the exam accounts for 50% of your overall exam score. 

Section II of the AP Comparative Government exam consists of four free-response questions . On this part of the exam, you’ll be asked to provide open-ended, written responses to all four free-response questions. Section II lasts for one hour and 30 minutes and counts for 50% of your overall exam score . 

To give you a clearer picture of how the AP Comparative Government exam is structured, we’ve broken the core exam elements down in the table below:  

Source: The College Board

The AP Comparative Government and Politics exam tests you on a wide range of topics and skills that you need to really drive home before exam day. To help you prepare, we’ll go over the AP Comparative Government course themes, skills, and units next!

body-skills-training-cc0

What’s on the AP Government — Comparative Exam? Course Themes, Skills, and Units

The AP Government — Comparative course teaches you the skills used by political scientists . To develop these skills during the course, you’ll explore content that falls into five big ideas that guide the course. 

The five big ideas for AP Comparative Government are: 

  • Big Idea 1: Power and Authority
  • Big Idea 2: Legitimacy and Stability
  • Big Idea 3: Democratization
  • Big Idea 4: Internal/External Forces
  • Big Idea 5: Methods of Political Analysis 

On the AP Comparative Government exam, you’ll show your mastery of the skills associated with these big ideas by answering questions that ask you to apply concepts, analyze data, compare countries, and write political science arguments.

The content and skills you’ll study throughout the AP Comparative Government course are divided out into five units of study . You’ll be tested on content from all five course units during the AP Comparative Government exam. Getting familiar with what each unit covers and how those topics are weighted in your overall exam score will help you get prepared for exam day!

You can view each course unit, the topics they cover, and how they’re weighted in your exam score below: 

Now that you know what’s on the AP Comparative Government exam, let’s break down the two sections of the exam even further. We’ll look at Section I and Section II of the AP Comparative Government exam next!

AP Comparative Government Exam: Section I

The first section of the exam tests your ability to describe, explain, compare, and analyze political concepts and processes, various forms of data, and text passages. You’ll be asked to demonstrate these skills by answering both individual and sets of multiple-choice questions. 

Section I consists of 55 multiple-choice questions, lasts for one hour, and counts for 50% of your exam score. 

Here’s a breakdown of how each skill is assessed on the multiple-choice section of the exam: 

  • Approximately 40–55% of multiple-choice questions assess students’ ability to apply political concepts and processes in hypothetical and authentic contexts. 
  • Approximately 25–32% of multiple-choice questions will assess students’ ability to compare the political concepts and processes of China, Iran, Mexico, Nigeria, Russia, and the United Kingdom.
  • Approximately 10–16% of multiple-choice questions will assess students’ ability to analyze and interpret quantitative data represented in tables, charts, graphs, maps, and infographics
  • Approximately 9–11% of multiple-choice questions will assess students’ ability to read, analyze, and interpret text-based sources.

To help you get a better idea of what the multiple-choice questions are like on this part of the AP Comparative Government exam, let’s look at a sample question and how it’s scored next . 

Sample Question: Multiple-Choice

Looking at sample multiple-choice questions can help you grasp the connection between what you learn in the AP Comparative Government course and what you’ll be tested on during the exam. 

The individual multiple-choice question below comes from the College Board’s official guide to AP Comparative Government and Politics . 

body-ap-comp-gov-image-3

The multiple-choice question above asks you to compare two or more countries based on their political systems and behaviors. It draws on your knowledge of Big Idea #1: Power and Authority because it asks about the role of government executives in different countries . You’ll focus on these concepts during Unit 2 of your AP Comparative Government course, which explores political institutions in different countries. 

The correct answer to this multiple-choice question is D : “The Chinese president and the Nigerian president are both commanders in chief of the armed forces.” 

AP Comparative Government Exam: Section II

Like Section I, the second section of the exam tests your ability to describe, explain, compare, and analyze political concepts and processes, various forms of data, and text passages. In this section, you’ll be asked to demonstrate these skills by providing written responses . 

Section II consists of four free-response questions, lasts for one hour and 30 minutes, and counts for 50% of your exam score. 

There are four different types of free-response questions on the exam, and each one tests your reading and writing skills in different ways. Here’s a breakdown of what you’ll be asked to do on each free-response question on the exam: 

  • 1 conceptual analysis question: You’ll define or describe a political concept and/or compare political systems, principles, institutions, processes, policies, or behaviors.
  • 1 quantitative analysis question: You’ll analyze data to find patterns and trends and reach a conclusion.
  • 1 comparative analysis question: You’ll compare political concepts, systems, institutions, processes, or policies in two of the course countries.
  • 1 argument essay: You’ll write an evidence-based essay supporting a claim or thesis.

To help you get a better sense of what the free-response questions are like on this part of the AP Comparative Government exam, let’s look at an example of each type of question and how it’s scored next . 

Sample Question: Conceptual Analysis Free-Response

The free-response question below is taken from the College Board’s official guide to AP Comparative Government and Politics . This sample question is an example of a conceptual analysis question. This is the first type of question that you’ll encounter on the exam. 

On the real exam, you’ll have 10 minutes to answer the conceptual analysis question . Check out the question below: 

body-ap-comp-gov-image-1

To understand how to answer this question correctly, we’ll need to look at how conceptual analysis questions are scored on the exam. The scoring rubric below shows how your response to this question would be evaluated after the exam: 

Sample Question: Quantitative Analysis Free-Response

The Quantitative Analysis free-response question gives you quantitative data in the form of a graph, table, map, or infographic. You’ll be asked to describe, draw a conclusion, or explain that data and its connections to key course concepts. 

The quantitative analysis question is the second question you’ll encounter on the exam. It’s worth five raw points of your score on this section of the exam, and you should spend about 20 minutes answering this question. 

The quantitative analysis question below comes from the College Board’s official guide to AP Comparative Government and Politics : 

body-ap-comp-gov-image-4

To get a better idea of how to answer this question, let’s look at the scoring rubric that’s used to evaluate this quantitative analysis question on the exam: 

Sample Question: Comparative Analysis Free-Response

The Comparative Analysis free-response question assesses your ability to define, describe, compare, or explain political concepts, systems, institutions, or policies in different countries. This question is the third free-response question that you’ll answer on the exam. 

The Comparative Analysis question is worth five raw points of your score on this section of the exam, and you should spend about 20 minutes answering this question. 

The comparative analysis question below comes from the College Board’s official guide to AP Comparative Government and Politics : 

body-ap-comp-gov-image-6

We can take a look at the scoring rubric that’s used to evaluate this type of free-response question to get a better idea of what types of responses will earn you full points: 

Sample Question: Argument Essay Free-Response

The fourth and final free-response question you’ll encounter on the exam is the Argument Essay question. This free-response question assesses your ability to make a claim that responds to the question, defend and support your claim with reasonable evidence, and respond to an opposing view on the topic at hand. 

The Argument Essay question is worth five raw points, and it’s recommended that you spend about 40 minutes answering this question. 

The argument essay question below comes from the College Board’s official guide to AP Comparative Government and Politics : 

body-ap-comp-gov-image-2

To understand what an effective response to this question looks like, we’ll need to think about how argument essay questions are scored on the exam. 

The scoring rubric for this free-response question is quite long; you’ll find four separate categories for evaluation in the rubric below , as well as examples of responses that will earn you full points in each category. 

The scoring rubric below shows how your response to this question will be evaluated: 

 

How Is the AP Comparative Government Exam Scored? 

Before you take the AP Comparative Government exam, you need to know how your responses will be scored. Here, we’ll explain how each section of the AP Comparative Government exam is scored, scaled, and combined to produce your final score on the AP 1-5 scale .

As a quick reminder, here’s how the score percentages breakdown on the exam: 

  • Section I: Multiple-choice: 55 questions, 50% of overall score
  • Section II: Free-response: four questions, 50% of overall score
  • Question 1: Conceptual Analysis: 11%
  • Question 2: Quantitative Analysis: 12.5%
  • Question 3: Comparative Analysis: 12.5%
  • Question 4: Argument Essay: 14%

On the multiple-choice section, you’ll earn one raw point for each question you answer correctly. The maximum number of raw points you can earn on the multiple-choice section is 55 points. You won’t lose any points for incorrect answers!

The free-response questions are scored differently. The Conceptual Analysis question is worth four raw points, and the Quantitative Analysis, Comparative Analysis, and Argument Essay questions are each worth five raw points. Collectively, there are a total of 19 raw points you can earn on the free-response section . 

Remember: you’ll only lose points on free-response questions for big errors , like providing an incorrect definition or failing to justify your reasoning. While you should use proper grammar and punctuation, you won’t be docked points for minor errors as long as your responses are clear and easy to understand. 

You can earn 74 raw points on the AP Comparative Government exam. Here’s how those points are parsed out by section: 

  • 55 points for multiple-choice
  • 19 points for free-response

After your raw scores have been tallied, the College Board will convert your raw score into a scaled score of 1-5 . When you receive your score report, that 1-5 scaled score is the one you’ll see. 

The 5 rate for the AP Comparative Government exam is fairly middle-of-the-road in comparison to other AP exams . Take a look at the table below to see what percentage of test takers earned each possible scaled score on the 2021 AP Comparative Government exam: 

body-number-four-post-it-note

4 Top Tips for Prepping for the AP Comparative Government and Politics Exam

If the AP Comparative Government exam is right around the corner for you, you’re probably thinking about how to prepare! We’re here to help you with that. C heck out our four best tips for studying for the AP Comparative Government exam !

Tip 1: Start With a Practice Exam

One of the best ways to set yourself up for successful AP exam prep is to take a practice exam. Taking a practice AP Comparative Government exam before you really start studying can help you design a study routine that best suits your needs. 

When you take a practice exam before diving into your study regimen, you get the chance to identify your strengths and weaknesses. Identifying your weaknesses early on in your exam prep will help you tailor your study time to eliminating your weaknesses (which translates to earning more points on the exam!). 

We recommend taking a full practice exam in the time frame you’ll be allotted on the real exam. This will help you get a real sense of what the timing will feel like on exam day! After you take the practice exam, sit down and evaluate your results. Make note of the questions you missed, the skills those questions assess, and the course content they reference. You can then design a study routine that targets those tougher areas–and give yourself a better chance of earning full exam points in the process!

Tip 2: Create Your Own Cram Sheet

Everyone needs quality study materials in order to prepare well for AP exams. But did you know that creating your own study materials is a great way to help you remember tough material? Creating your own AP Comparative Government cram sheet is a great way to review course concepts and themes and organize your understanding of the material you’ll be tested over later.

You can look up AP Comparative Government cram sheets online and design yours in a similar way…or you can take some time to consider your needs as a learner and test-taker, then design a cram sheet that’s tailor-made for you. 

On your cram sheet, you’ll likely want to include course concepts, issues, and questions that pop up on homework, quizzes, and tests that you take in your AP Comparative Government class. From there, you can supplement your cram sheet with info you learn from practice exams, sample free-response questions, and official scoring rubrics. You can work on memorizing that material, or simply use it to organize your study routine!

Tip 3: Practice Free-Response Questions

Free-response questions on AP exams are notoriously difficult, and the AP Government Comparative free-response questions are no different. Writing-based questions can be intimidating for any test-taker, so it’s important to practice free-response questions before the exam. 

The College Board provides an archive of past official free-response questions on their website . You can use these to practice and study! Any free-response questions your teacher gives you in class are fair game as well. When you practice free-response questions, remember to stick to the timing you’ll be given on the real exam, and use official scoring rubrics to evaluate your responses. Doing these things will help you get used to what free-response questions will feel like on the real exam! 

Tip 4: Take Another Practice Exam

As you wrap up your exam prep and exam day nears, consider taking another practice exam. You can compare your results on your second practice exam to your results on the practice exam that you took before you started studying. You’ll get to see how much you’ve improved over time!

Taking a final practice exam a few weeks before exam day can also help you revamp your exam prep. You can use your exam results to focus your final study time on any remaining struggle areas you’re encountering. Also, your score on your final practice exam can help you get an idea of what you’re likely to score on the real exam. Having this knowledge going into test day can calm your nerves and give you confidence, which are both essential to success on the AP Comparative Government exam!

body_next

What's Next? 

If you're taking AP Comparative Government, you're probably thinking about taking more AP classes during high school. Here's a list of the hardest AP classes and tests for you. 

Wondering how your AP Comparative Government score stacks up to the competition? Here's a list of the average AP scores for  every  exam to help you figure out. 

If you want to get a 5 on your AP exams, you'll need a study plan. Our five-step AP study plan will help you study smarter and boost your scores. 

Looking for help studying for your AP exam? Our one-on-one online AP tutoring services can help you prepare for your AP exams. Get matched with a top tutor who got a high score on the exam you're studying for!

Ashley Sufflé Robinson has a Ph.D. in 19th Century English Literature. As a content writer for PrepScholar, Ashley is passionate about giving college-bound students the in-depth information they need to get into the school of their dreams.

Ask a Question Below

Have any questions about this article or other topics? Ask below and we'll reply!

Improve With Our Famous Guides

  • For All Students

The 5 Strategies You Must Be Using to Improve 160+ SAT Points

How to Get a Perfect 1600, by a Perfect Scorer

Series: How to Get 800 on Each SAT Section:

Score 800 on SAT Math

Score 800 on SAT Reading

Score 800 on SAT Writing

Series: How to Get to 600 on Each SAT Section:

Score 600 on SAT Math

Score 600 on SAT Reading

Score 600 on SAT Writing

Free Complete Official SAT Practice Tests

What SAT Target Score Should You Be Aiming For?

15 Strategies to Improve Your SAT Essay

The 5 Strategies You Must Be Using to Improve 4+ ACT Points

How to Get a Perfect 36 ACT, by a Perfect Scorer

Series: How to Get 36 on Each ACT Section:

36 on ACT English

36 on ACT Math

36 on ACT Reading

36 on ACT Science

Series: How to Get to 24 on Each ACT Section:

24 on ACT English

24 on ACT Math

24 on ACT Reading

24 on ACT Science

What ACT target score should you be aiming for?

ACT Vocabulary You Must Know

ACT Writing: 15 Tips to Raise Your Essay Score

How to Get Into Harvard and the Ivy League

How to Get a Perfect 4.0 GPA

How to Write an Amazing College Essay

What Exactly Are Colleges Looking For?

Is the ACT easier than the SAT? A Comprehensive Guide

Should you retake your SAT or ACT?

When should you take the SAT or ACT?

Stay Informed

Get the latest articles and test prep tips!

Follow us on Facebook (icon)

Looking for Graduate School Test Prep?

Check out our top-rated graduate blogs here:

GRE Online Prep Blog

GMAT Online Prep Blog

TOEFL Online Prep Blog

Holly R. "I am absolutely overjoyed and cannot thank you enough for helping me!”

ap gov argument essay example 2021

  • AP Calculus
  • AP Chemistry
  • AP U.S. History
  • AP World History
  • Free AP Practice Questions
  • AP Exam Prep

AP U.S. Government and Politics: Concept Application Example

Concept application strategy.

  • When analyzing the stimulus, carefully note relevant details. Paraphrase the political scenario in your own words before looking at the questions.
  • Concept Application questions often build on each other, asking you to use your response for one part to answer another part. Therefore, carefully plan your response before you start writing in order to make sure you choose answers that you can apply to later parts of the prompt if needed.

Sample Concept Application Question

A new political party, the Health & Wealth Party, forms to focus on those policies which members believe will address the most significant threats to the health and prosperity of the general population. Their key platform favors requiring manufacturers of high-sugar snack foods to produce an equal ratio of low-sugar snack alternatives offered at the same price to consumers. In addition, they advocate using tax money to subsidize low-income families with funds to buy the low-sugar snacks. To increase “wealth,” the party supports significantly lowering taxes on corporations, with the intent of attracting new businesses to the United States. Finally, they also propose drastically reducing income taxes for all Americans, making up the difference in the budget by slashing military spending for foreign affairs. After reading the scenario, respond to Parts A, B, and C. (A) The Health & Wealth Party’s platform contains elements that reflect conservative, liberal, and libertarian viewpoints. For each of the three viewpoints, identify one element of the Health & Wealth Party platform that reflects that viewpoint’s typical ideology. (B) Describe a way in which the Health & Wealth Party’s nominee, if elected to the presidency, could attempt to implement the policy regulating snack manufacturers. (C) Explain one reason why it is difficult for third-party candidates, like the Health & Wealth Party nominee, to win presidential elections.

Step 1: Analyze the Prompt

Step 2: plan your response.

  • Conserv.: traditional values + pro-market policies –> lower taxes on business
  • Liberal: more gov’t involvement for equality –> regulate manuf. & subsidies for poor
  • Libert.: ind. liberty + low gov’t involvement –> lower income taxes & military spending

Part B: Need to think of how a pres. could impact a policy about reg. snack manufacturers, including details to describe my answer.

  • Commerce clause is relevant
  • Pres. could meet w/ Congress, persuade to make committee & draft bill
  • Pres. could endorse candidates who agree on issue

Part C: Need to think about difficulties faced by third-party candidates during elections (not while in office). Need to fully describe the issue, including the why/how, to count as explaining.

  • Hard to win votes in electoral college due to winner-take-all system and entrenchment of major parties
  • But an election based only on popular vote would likely not create a clear majority winner

Step 3: Action! Write Your Response

Step 4: proofread, sample high-scoring response, sample response explanation.

  • Organization: The response addresses one part in each paragraph. Although this is not required, it makes it much easier for the reader to follow and score your response.
  • Sentences: Although Part A requires only identification, the writer still uses a paragraph for the response, adding just a little explanation to justify his or her classifications of the party planks. Use paragraphs and complete sentences for all parts of your responses; never use just phrases or lists.
  • Addressing each action word: Note that the responses for Parts B and C are longer than the response for Part A. Part A only required identification, while B required description and C required explanation. The response for Part B provides a full description of a presidential action. The response for Part C effectively explains by discussing multiple reasons why the Electoral College is the way it is, including both how the system puts third-party candidates at a disadvantage and why the system is still practical.

Scoring for Question 1: 3 points (1 + 1 + 1)

  • Example conservative components: lowering taxes on businesses, lowering income taxes, seeking more balanced budget
  • Example liberal components: regulating snack manufacturers, providing subsidies for lower- income families, lowering military spending
  • Example libertarian components: lowering taxes on businesses, lowering income taxes, lowering foreign military spending, seeking more balanced budget

Part B (1 point) One point for describing a way the president could impact policy.

  • Example ways include: calling a special committee/commission to research and influence the issue, persuading Congress members to create legislation that addresses the policy, appointing positions to the Food and Drug Administration that support the policy, issuing an executive order to the FDA, endorsing candidates who support the policy, using the “bully pulpit” to rally public support and put pressure on Congress, highlighting the issue in the State of the Union address, proposing a budget that includes provisions for the policy, threatening to veto a bill unless Congress makes provisions for the policy

Part C (1 point) One point for explaining a difficulty faced by third-party candidates.

  • Example difficulties include: less financing, difficulty of getting onto ballots, heavy political entrenchment of the two-party system, winner-take-all nature of Electoral College makes it difficult to score electoral votes, voter discouragement (wanting to make sure their vote “counts”), major parties’ tendency to adopt platform planks that try to appeal to potential third-party voters

Learn more about the other free response questions on the AP U.S. Government and Politics exam. Quantitative Analysis   •   SCOTUS Comparison   •   Argument Essay

You might also like

AP US Government and Politics Argument Essay

Call 1-800-KAP-TEST or email [email protected]

Prep for an Exam

MCAT Test Prep

LSAT Test Prep

GRE Test Prep

GMAT Test Prep

SAT Test Prep

ACT Test Prep

DAT Test Prep

NCLEX Test Prep

USMLE Test Prep

Courses by Location

NCLEX Locations

GRE Locations

SAT Locations

LSAT Locations

MCAT Locations

GMAT Locations

Useful Links

Kaplan Test Prep Contact Us Partner Solutions Work for Kaplan Terms and Conditions Privacy Policy CA Privacy Policy Trademark Directory

Are you seeking one-on-one college counseling and/or essay support? Limited spots are now available. Click here to learn more.

How to Write the AP Lang Argument Essay (With Example)

December 14, 2023

ap lang argument essay example

We’d like to let you in on a little secret: no one, including us, enjoys writing timed essays. But a little practice goes a long way. If you want to head into your AP English Exam with a cool head, you’ll want to know what you’re getting into ahead of time. We can’t promise the AP Lang Argument Essay will ever feel like an island vacation, but we do have tons of hand tips and tricks (plus a sample essay!) below to help you do your best. This article will cover: 1) What is the AP Lang Argumentative Essay? 2) AP Lang Argument Rubric 3) AP Lang Argument Sample Prompt 4) AP Lang Argument Essay Example 5) AP Lang Argument Essay Example: Answer Breakdown.

What is the AP Lang Argument Essay?

The AP Lang Argument Essay is one of three essays included in the written portion of the AP English Exam. The full AP English Exam is 3 hours and 15 minutes long, with the first 60 minutes dedicated to multiple-choice questions. Once you complete the multiple-choice section, you move on to three equally weighted essays that ask you to synthesize, analyze, and interpret texts and develop well-reasoned arguments. The three essays include:

Synthesis essay: You’ll review various pieces of evidence and then write an essay that synthesizes (aka combines and interprets) the evidence and presents a clear argument. Read our write-up on How to Write the AP Lang Synthesis Essay here.

Argumentative essay: You’ll take a stance on a specific topic and argue your case.

Rhetorical essay: You’ll read a provided passage, then analyze the author’s rhetorical choices and develop an argument that explains why the author made those rhetorical choices. Read our write-up on How to Write the AP Lang Rhetorical Essay here.

AP Lang Argument Essay Rubric

The AP Lang Argument Essay is graded on 3 rubric categories : Thesis, Evidence and Commentary, and Sophistication . How can you make sure you cover all three bases in your essay? We’ll break down each rubric category with dos and don’ts below:

  • Thesis (0-1 point)

When it comes to grading your thesis, AP Exam graders are checking off a box: you either have a clear thesis or you don’t. So, what crucial components of a thesis will get you your check mark?

  • Make sure your thesis argues something . To satisfy your graders, your thesis needs to take a clear stance on the issue at hand.
  • Include your thesis statement in your intro paragraph. The AP Lang Argumentative essay is just that: an essay that makes an argument, so make sure you present your argument right away at the end of your first paragraph.
  • A good test to see if you have a thesis that makes an argument for your AP Lang Argumentative Essay: In your head, add the phrase “I agree/disagree that…” to the beginning of your thesis. If what follows doesn’t logically flow after that phrase (aka if what follows isn’t an agreement or disagreement), it’s likely you’re not making an argument.
  • In your thesis, outline the evidence you’ll cover in your body paragraphs.

AP Lang Argument Essay Rubric (Continued)

  • Avoid a thesis that merely restates the prompt.
  • Avoid a thesis that summarizes the text but does not make an argument.
  • Avoid a thesis that weighs the pros and cons of an issue. Your job in your thesis is to pick a side and stick with it.
  • Evidence and Commentary (0-4 points)

This rubric category is graded on a scale of 0-4 where 4 is the highest grade. Unlike the rhetorical and synthesis essays, the evidence you need to write your AP Lang Argument Essay is not provided to you. Rather, you’ll need to generate your own evidence and comment upon it.

What counts as evidence?

Typically, the AP Lang Argument Essay prompt asks you to reflect on a broad cultural, moral, or social issue that is open to debate. For evidence, you won’t be asked to memorize and cite statistics or facts. Rather, you’ll want to bring in real-world examples of:

  • Historical events
  • Current-day events from the news
  • Personal anecdotes

For this essay, your graders know that you’re not able to do research to find the perfect evidence. What’s most important is that you find evidence that logically supports your argument.

What is commentary?

In this essay, it’s important to do more than just provide examples relevant evidence. After each piece of evidence you include, you’ll need to explain why it’s significant and how it connects to your main argument. The analysis you include after your evidence is commentary .

  • Take a minute to brainstorm evidence that logically supports your argument. If you have to go out of your way to find the connection, it’s better to think of different evidence.
  • Include multiple pieces of evidence. There is no magic number, but do make sure you incorporate more than a couple pieces of evidence that support your argument.
  • Make sure you include more than one example of evidence, too. Let’s say you’re working on an essay that argues that people are always stronger together than apart. You’ve already included an example from history: during the civil rights era, protestors staged group sit-ins as a powerful form of peaceful protest. That’s just one example, and it’s hard to make a credible argument with just one piece of evidence. To fix that issue, think of additional examples from history, current events, or personal experience that are not related to the civil rights era.
  • After you include each piece of evidence, explain why it’s significant and how it connects to your main argument.
  • Don’t summarize or speak generally about the topic. Everything you write must be backed up with specific and relevant evidence and examples.
  • Don’t let quotes speak for themselves. After every piece of evidence you include, make sure to explain and connect the evidence to your overarching argument.

AP Lang Argument Essay (Continued)

  • Sophistication (0-1 point)

According to the College Board , one point can be awarded to AP Lang Argument essays that achieve a high level of sophistication. You can accomplish that in four ways:

  • Crafting a nuanced argument by consistently identifying and exploring complexities or tensions.
  • Articulating the implications or limitations of an argument by situating it within a broader context.
  • Making effective rhetorical choices that consistently strengthen the force and impact of the student’s argument.
  • Employing a style that is consistently vivid and persuasive.

In sum, this means you can earn an additional point for going above and beyond in depth, complexity of thought, or by writing an especially persuasive, clear, and well-structured essay. In order to earn this point, you’ll first need to do a good job with the fundamentals: your thesis, evidence, and commentary. Then, to earn your sophistication point, follow these tips:

  • Outline your essay before you begin to ensure it flows in a clear and cohesive way.
  • Include well-rounded evidence. Don’t rely entirely on personal anecdotes, for example. Incorporate examples from current events or history, as well.
  • Thoroughly explain how each piece of evidence connects to your thesis in order to fully develop your argument.
  • Explore broader implications. If what you’re arguing is true, what does that mean to us today? Who is impacted by this issue? What real-world issues are relevant to this core issue?
  • Briefly explore the other side of the issue. Are the instances where your argument might not be true? Acknowledge the other side, then return to proving your original argument.
  • Steer clear of generalizations (avoid words like “always” and “everyone”).
  • Don’t choose an argument you can’t back up with relevant examples.
  • Avoid complex sentences and fancy vocabulary words unless you use them often. Long, clunky sentences with imprecisely used words are hard to follow.

AP Lang Argument Sample Prompt

The sample prompt below is published online by the College Board and is a real example from the 2021 AP English Exam. The prompt provides background context, essay instructions, and the text you need to analyze.

Suggested time—40 minutes.

Many people spend long hours trying to achieve perfection in their personal or professional lives. Similarly, people often demand perfection from others, creating expectations that may be challenging to live up to. In contrast, some people think perfection is not attainable or desirable.

Write an essay that argues your position on the value of striving for perfection.

In your response you should do the following:

  • Respond to the prompt with a thesis that presents a defensible position.
  • Provide evidence to support your line of reasoning.
  • Explain how the evidence supports your line of reasoning.
  • Use appropriate grammar and punctuation in communicating your argument.

AP Lang Argument Essay Example

As the old phrase says, “Practice makes perfect.” But is perfection something that is actually attainable? Sometimes, pushing for perfection helps us achieve great things, but most often, perfectionism puts too much pressure on us and prevents us from knowing when we have done the best we can. Striving for perfection can only lead us to shortchange ourselves. Instead, we should value learning, growth, and creativity and not worry whether we are first or fifth best.

Students often feel the need to be perfect in their classes, and this can cause students to struggle or stop making an effort in class. In elementary and middle school, for example, I was very nervous about public speaking. When I had to give a speech, my voice would shake, and I would turn very red. My teachers always told me “relax!” and I got Bs on Cs on my speeches. As a result, I put more pressure on myself to do well, spending extra time making my speeches perfect and rehearsing late at night at home. But this pressure only made me more nervous, and I started getting stomach aches before speaking in public.

Once I got to high school, however, I started doing YouTube make-up tutorials with a friend. We made videos just for fun, and laughed when we made mistakes or said something silly. Only then, when I wasn’t striving to be perfect, did I get more comfortable with public speaking.

AP Lang Argumentative Essay Example (Continued)

In the world of art and business and science, perfectionism can also limit what we are able to achieve. Artists, for example, have to take risks and leave room for creativity. If artists strive for perfection, then they won’t be willing to fail at new experiments and their work will be less innovative and interesting. In business and science, many products, like penicillin for example, were discovered by accident. If the scientist who discovered penicillin mold growing on his petri dishes had gotten angry at his mistake and thrown the dishes away, he would never have discovered a medicine that is vital to us today.

Some fields do need to value perfection. We wouldn’t like it, for example, if our surgeon wasn’t striving for perfection during our operation. However, for most of us, perfectionism can limit our potential for learning and growth. Instead of trying to be perfect, we should strive to learn, innovate, and do our personal best.

AP Lang Argument Essay Example: Answer Breakdown

The sample AP Lang Argumentative Essay above has some strengths and some weaknesses. Overall, we would give this essay a 3 or a 4. Let’s break down what’s working and what could be improved:

  • The essay offers a thesis that makes a clear argument that is relevant to the prompt: “Striving for perfection can only lead us to shortchange ourselves. Instead, we should value learning, growth, and creativity and not worry whether we are first or fifth best.”
  • The first body paragraph provides evidence that supports the essay’s thesis. This student’s personal anecdote offers an example of a time when perfectionism led them to shortchange themselves.
  • The second body paragraph provides additional evidence that supports the essay’s thesis. The example describing the discovery of penicillin offers another example of a situation in which perfectionism might have limited scientific progress.
  • The writer offers commentary explaining how her examples of public speaking and penicillin illustrate that we should “value learning, growth, and creativity” over perfectionism.
  • The essay follows one line of reasoning and does not stray into tangents.
  • The essay is organized well with intro, body, and concluding paragraphs. Overall, it is easy to read and is free of grammar errors.

What could be improved:

  • Although the second body paragraph provides one good specific example about the discovery of penicillin, the other examples it offers about art and business are only discussed generally and aren’t backed up with evidence. This paragraph would be stronger if it provided more examples. Or, if this writer couldn’t think of examples, they could have left out mentions of art and business altogether and included alternate evidence instead.
  • This writer would more thoroughly support their argument if they were able to offer one more example of evidence. They could provide another personal anecdote, an example from history, or an example from current events.
  • The writer briefly mentions the other side of the argument in their concluding paragraph: “Some fields do need to value perfection. We wouldn’t like it, for example, if our surgeon wasn’t striving for perfection during our operation.” Since it’s so brief a mention of the other side, it undermines the writer’s overall argument. This writer should either dedicate more time to reflecting on why even surgeons should “value learning, growth, and creativity” over perfectionism, or they should leave these sentences out.

AP Lang Argument Essay Example—More Resources

Looking for more tips to help you master your AP Lang Argumentative Essay? Brush up on 20 Rhetorical Devices High School Students Should Know and read our Tips for Improving Reading Comprehension .

If you’re ready to start studying for another part of the AP English Exam, find more expert tips in our How to Write the AP Lang Synthesis and How to Write the AP Lang Rhetorical Essay blog posts.

  • High School Success

' src=

Christina Wood

Christina Wood holds a BA in Literature & Writing from UC San Diego, an MFA in Creative Writing from Washington University in St. Louis, and is currently a Doctoral Candidate in English at the University of Georgia, where she teaches creative writing and first-year composition courses. Christina has published fiction and nonfiction in numerous publications, including The Paris Review , McSweeney’s , Granta , Virginia Quarterly Review , The Sewanee Review , Mississippi Review , and Puerto del Sol , among others. Her story “The Astronaut” won the 2018 Shirley Jackson Award for short fiction and received a “Distinguished Stories” mention in the 2019 Best American Short Stories anthology.

  • 2-Year Colleges
  • Application Strategies
  • Best Colleges by Major
  • Best Colleges by State
  • Big Picture
  • Career & Personality Assessment
  • College Essay
  • College Search/Knowledge
  • College Success
  • Costs & Financial Aid
  • Data Visualizations
  • Dental School Admissions
  • Extracurricular Activities
  • Graduate School Admissions
  • High Schools
  • Homeschool Resources
  • Law School Admissions
  • Medical School Admissions
  • Navigating the Admissions Process
  • Online Learning
  • Outdoor Adventure
  • Private High School Spotlight
  • Research Programs
  • Summer Program Spotlight
  • Summer Programs
  • Teacher Tools
  • Test Prep Provider Spotlight

College Transitions Sidebar Block Image

“Innovative and invaluable…use this book as your college lifeline.”

— Lynn O'Shaughnessy

Nationally Recognized College Expert

College Planning in Your Inbox

Join our information-packed monthly newsletter.

I am a... Student Student Parent Counselor Educator Other First Name Last Name Email Address Zip Code Area of Interest Business Computer Science Engineering Fine/Performing Arts Humanities Mathematics STEM Pre-Med Psychology Social Studies/Sciences Submit

What are your chances of acceptance?

Calculate for all schools, your chance of acceptance.

Duke University

Your chancing factors

Extracurriculars.

ap gov argument essay example 2021

How to Write the AP Lang Argument Essay + Examples

What’s covered:, what is the ap language argument essay, tips for writing the ap language argument essay, ap english language argument essay examples, how will ap scores impact my college chances.

In 2023, over 550,148 students across the U.S. took the AP English Language and Composition Exam, and 65.2% scored higher than a 3. The AP English Language Exam tests your ability to analyze a piece of writing, synthesize information, write a rhetorical essay, and create a cohesive argument. In this post, we’ll be discussing the best way to approach the argumentative essay section of the test, and we’ll give you tips and tricks so you can write a great essay.

The AP English Language Exam as of 2023 is structured as follows:

Section 1: 45 multiple choice questions to be completed in an hour. This portion counts for 45% of your score. This section requires students to analyze a piece of literature. The questions ask about its content and/or what could be edited within the passage.

Section 2: Three free response questions to be completed in the remaining two hours and 15 minutes. This section counts for 55% of your score. These essay questions include the synthesis essay, the rhetorical essay, and the argumentative essay.

  • Synthesis essay: Read 6-7 sources and create an argument using at least three of the sources.
  • Rhetorical analysis essay: Describe how a piece of writing evokes meaning and symbolism.
  • Argumentative essay: Pick a side of a debate and create an argument based on evidence. In this essay, you should develop a logical argument in support of or against the given statement and provide ample evidence that supports your conclusion. Typically, a five paragraph format is great for this type of writing. This essay is scored holistically from 1 to 9 points.

Do you want more information on the structure of the full exam? Take a look at our in-depth overview of the AP Language and Composition Exam .

Although the AP Language Argument may seem daunting at first, once you understand how the essay should be structured, it will be a lot easier to create cohesive arguments.

Below are some tips to help you as you write the essay.

1. Organize your essay before writing

Instead of jumping right into your essay, plan out what you will say beforehand. It’s easiest to make a list of your arguments and write out what facts or evidence you will use to support each argument. In your outline, you can determine the best order for your arguments, especially if they build on each other or are chronological. Having a well-organized essay is crucial for success.

2. Pick one side of the argument, but acknowledge the other side

When you write the essay, it’s best if you pick one side of the debate and stick with it for the entire essay. All your evidence should be in support of that one side. However, in your introductory paragraph, as you introduce the debate, be sure to mention any merit the arguments of the other side has. This can make the essay a bit more nuanced and show that you did consider both sides before determining which one was better. Often, acknowledging another viewpoint then refuting it can make your essay stronger.

3. Provide evidence to support your claims

The AP readers will be looking for examples and evidence to support your argument. This doesn’t mean that you need to memorize a bunch of random facts before the exam. This just means that you should be able to provide concrete examples in support of your argument.

For example, if the essay topic is about whether the role of the media in society has been detrimental or not, and you argue that it has been, you may talk about the phenomenon of “fake news” during the 2016 presidential election.

AP readers are not looking for perfect examples, but they are looking to see if you can provide enough evidence to back your claim and make it easily understood.

4. Create a strong thesis statement

The thesis statement will set up your entire essay, so it’s important that it is focused and specific, and that it allows for the reader to understand your body paragraphs. Make sure your thesis statement is the very last sentence of your introductory paragraph. In this sentence, list out the key points you will be making in the essay in the same order that you will be writing them. Each new point you mention in your thesis should start a paragraph in your essay.

Below is a prompt and sample student essay from the May 2019 exam . We’ll look at what the student did well in their writing and where they could improve.

Prompt: “The term “overrated” is often used to diminish concepts, places, roles, etc. that the speaker believes do not deserve the prestige they commonly enjoy; for example, many writers have argued that success is overrated, a character in a novel by Anthony Burgess famously describes Rome as a “vastly overrated city,” and Queen Rania of Jordan herself has asserted that “[b]eing queen is overrated.”

Select a concept, place, role, etc. to which you believe that the term “overrated” should be applied. Then, write a well-developed essay in which you explain your judgment. Use appropriate evidence from your reading, experience, or observations to support your argument.

Sample Student Essay #1:

[1] Competition is “overrated.” The notion of motivation between peers has evolved into a source of unnecessary stress and even lack of morals. Whether it be in an academic environment or in the industry, this new idea of competition is harmful to those competing and those around them.

[2] Back in elementary school, competition was rather friendly. It could have been who could do the most pushups or who could get the most imaginary points in a classroom for a prize. If you couldn’t do the most pushups or win that smelly sticker, you would go home and improve yourself – there would be no strong feelings towards anyone, you would just focus on making yourself a better version of yourself. Then as high school rolled around, suddenly applying for college doesn’t seem so far away –GPA seems to be that one stat that defines you – extracurriculars seem to shape you – test scores seem to categorize you. Sleepless nights, studying for the next day’s exam, seem to become more and more frequent. Floating duck syndrome seems to surround you (FDS is where a competitive student pretends to not work hard but is furiously studying beneath the surface just like how a duck furiously kicks to stay afloat). All of your competitors appear to hope you fail – but in the end what do you and your competitor’s gain? Getting one extra point on the test? Does that self-satisfaction compensate for the tremendous amounts of acquired stress? This new type of “competition” is overrated – it serves nothing except a never-ending source of anxiety and seeks to weaken friendships and solidarity as a whole in the school setting.

[3] A similar idea of “competition” can be applied to business. On the most fundamental level, competition serves to be a beneficial regulator of prices and business models for both the business themselves and consumers. However, as businesses grew increasingly greedy and desperate, companies resorted to immoral tactics that only hurt their reputations and consumers as a whole. Whether it be McDonald’s coupons that force you to buy more food or tech companies like Apple intentionally slowing down your iPhone after 3 years to force you to upgrade to the newest device, consumers suffer and in turn speak down upon these companies. Similar to the evolved form of competition in school, this overrated form causes pain for all parties and has since diverged from the encouraging nature that the principle of competition was “founded” on.

The AP score for this essay was a 4/6, meaning that it captured the main purpose of the essay but there were still substantial parts missing. In this essay, the writer did a good job organizing the sections and making sure that their writing was in order according to the thesis statement. The essay first discusses how competition is harmful in elementary school and then discusses this topic in the context of business. This follows the chronological order of somebody’s life and flows nicely.

The arguments in this essay are problematic, as they do not provide enough examples of how exactly competition is overrated. The essay discusses the context in which competition is overrated but does not go far enough in explaining how this connects to the prompt.

In the first example, school stress is used to explain how competition manifests. This is a good starting point, but it does not talk about why competition is overrated; it simply mentions that competition can be unhealthy. The last sentence of that paragraph is the main point of the argument and should be expanded to discuss how the anxiety of school is overrated later on in life. 

In the second example, the writer discusses how competition can lead to harmful business practices, but again, this doesn’t reflect the reason this would be overrated. Is competition really overrated because Apple and McDonald’s force you to buy new products? This example could’ve been taken one step farther. Instead of explaining why business structures—such as monopolies—harm competition, the author should discuss how those particular structures are overrated.

Additionally, the examples the writer used lack detail. A stronger essay would’ve provided more in-depth examples. This essay seemed to mention examples only in passing without using them to defend the argument.

It should also be noted that the structure of the essay is incomplete. The introduction only has a thesis statement and no additional context. Also, there is no conclusion paragraph that sums up the essay. These missing components result in a 4/6.

Now let’s go through the prompt for a sample essay from the May 2022 exam . The prompt is as follows:

Colin Powell, a four-star general and former United States Secretary of State, wrote in his 1995 autobiography: “[W]e do not have the luxury of collecting information indefinitely. At some point, before we can have every possible fact in hand, we have to decide. The key is not to make quick decisions, but to make timely decisions.”

Write an essay that argues your position on the extent to which Powell’s claim about making decisions is valid. 

In your response you should do the following:

  • Respond to the prompt with a thesis that presents a defensible position. 
  • Provide evidence to support your line of reasoning. 
  • Explain how the evidence supports your line of reasoning. 
  • Use appropriate grammar and punctuation in communicating your argument.

Sample Student Essay #2:

Colin Powell, who was a four star general and a former United States Secretary of State. He wrote an autobiography and had made a claim about making decisions. In my personal opinion, Powell’s claim is true to full extent and shows an extremely valuable piece of advice that we do not consider when we make decisions.

Powell stated, “before we can have every possible fact in hand we have to decide…. but to make it a timely decision” (1995). With this statement Powell is telling the audience of his autobiography that it does not necessarily matter how many facts you have, and how many things you know. Being able to have access to everything possible takes a great amount of time and we don’t always have all of the time in the world. A decision has to be made with what you know, waiting for something else to come in while trying to make a decision whether that other fact is good or bad you already have a good amount of things that you know. Everyone’s time is valuable, including yours. At the end of the day the decision will have to be made and that is why it should be made in a “timely” manner.

This response was graded for a score of 2/6. Let’s break down the score to smaller points that signify where the student fell short.

The thesis in this essay is clearly outlined at the end of the first paragraph. The student states their agreement with Powell’s claim and frames the rest of their essay around this stance. The success in scoring here lies in the clear communication of the thesis and the direction the argument will take. It’s important to make the thesis statement concise, specific, and arguable, which the student has successfully done.

While the student did attempt to provide evidence to support their thesis, it’s clear that their explanation lacks specific detail and substance. They referenced Powell’s statement, but did not delve into how this statement has proven true in specific instances, and did not provide examples that could bring the argument to life.

Commentary is an essential part of this section’s score. It means explaining the significance of the evidence and connecting it back to the thesis. Unfortunately, the student’s commentary here is too vague and does not effectively elaborate on how the evidence supports their argument.

To improve, the student could use more concrete examples to demonstrate their point and discuss how each piece of evidence supports their thesis. For instance, they could discuss specific moments in Powell’s career where making a timely decision was more valuable than waiting for all possible facts. This would help illustrate the argument in a more engaging, understandable way.

A high score in the “sophistication” category of the grading rubric is given for demonstrating a complex understanding of the rhetorical situation (purpose, audience, context, etc.), making effective rhetorical choices, or establishing a line of reasoning. Here, the student’s response lacks complexity and sophistication. They’ve simply agreed with Powell’s claim and made a few general statements without providing a deeper analysis or effectively considering the rhetorical situation.

To increase sophistication, the student could explore possible counterarguments or complexities within Powell’s claim. They could discuss potential drawbacks of making decisions without all possible facts, or examine situations where timely decisions might not yield the best results. By acknowledging and refuting these potential counterarguments, they could add more depth to their analysis and showcase their understanding of the complexities involved in decision-making.

The student could also analyze why Powell, given his background and experiences, might have come to such a conclusion, thus providing more context and showing an understanding of the rhetorical situation.

Remember, sophistication in argumentation isn’t about using fancy words or complicated sentences. It’s about showing that you understand the complexity of the issue at hand and that you’re able to make thoughtful, nuanced arguments. Sophistication shows that you can think critically about the topic and make connections that aren’t immediately obvious.

Now that you’ve looked at an example essay and some tips for the argumentative essay, you know how to better prepare for the AP English Language and Composition Exam.

While your AP scores don’t usually impact your admissions chances , colleges do care a lot about your course rigor. So, taking as many APs as you can will certainly boost your chances! AP scores can be a way for high-performing students to set themselves apart, particularly when applying to prestigious universities. Through the process of self-reporting scores , you can show your hard work and intelligence to admissions counselors.

That said, the main benefit of scoring high on AP exams comes once you land at your dream school, as high scores can allow you to “test out” of entry-level requirements, often called GE requirements or distribution requirements. This will save you time and money.

To understand how your course rigor stacks up, check out CollegeVine’s free chancing engine . This resource takes your course rigor, test scores, extracurriculars, and more, to determine your chances of getting into over 1600 colleges across the country!

Related CollegeVine Blog Posts

ap gov argument essay example 2021

AP United States Government and Politics

Review the free-response questions from the 2024 ap exam., updates for 2023-24.

Starting in the 2023-24 school year, AP U.S. Government and Politics will have an updated course framework and instructional materials.

Exam Overview

Exam questions assess the course concepts and skills outlined in the course framework. For more information, download the  AP U.S. Government and Politics Course and Exam Description  (CED).

Encourage your students to visit the  AP U.S. Government and Politics student page  for exam information.

Mon, May 6, 2024

AP U.S. Government and Politics Exam

Exam format.

The AP U.S. Government and Politics Exam has consistent question types, weighting, and scoring guidelines every year, so you and your students know what to expect on exam day.

Section I: Multiple Choice

55 Questions | 1 Hour 20 Minutes | 50% of Exam Score

  • Individual questions (no stimulus): ~30
  • Quantitative Analysis : Analysis and application of quantitative-based source material
  • Qualitative Analysis : Analysis and application of text-based (primary and secondary) sources
  • Visual Analysis : Analysis and application of qualitative visual information

Section II: Free Response

4 Questions | 1 Hour 40 Minutes | 50% of Exam Score

  • Concept Application : Respond to a political scenario, describe and explain the effects of a political institution, behavior, or process
  • Quantitative Analysis : Analyze quantitative data, identify a trend or pattern, or draw a conclusion from a visual representation and explain how it relates to a political principle, institution, process, policy, or behavior
  • SCOTUS Comparison : Compare a nonrequired Supreme Court case with a required Supreme Court case, explaining how information from the required case is relevant to the nonrequired one
  • Argument Essay : Develop an argument in the form of an essay, using evidence from required foundational documents and course concepts

Exam Questions and Scoring Information

Ap u.s. government and politics: exam questions and scoring information.

View free-response questions and scoring information from this year's exam and past exams.

Score Reporting

Ap score reports for educators.

Access your score reports.

  • Board of Regents
  • Business Portal

Nysed Logo

New York State Education Department

New York State Education Department Logo

  • Commissioner
  • USNY Affiliates
  • Organization Chart
  • Building Tours
  • Program Offices
  • Rules & Regulations
  • Office of Counsel
  • Office of State Review
  • Freedom of Information (FOIL)
  • Governmental Relations
  • Adult Education
  • Bilingual Education
  • Career & Technical Education
  • Cultural Education
  • Diversity, Equity, and Inclusion
  • Early Learning
  • Educator Quality
  • Every Student Succeeds Act (ESSA)
  • Graduation Measures
  • Higher Education
  • High School Equivalency
  • Indigenous Education
  • My Brother's Keeper

Office of the Professions

  • P-12 Education
  • Special Education
  • Vocational Rehabilitation
  • Next Generation Learning Standards: ELA and Math
  • Office of Standards and Instruction
  • Diploma Requirements
  • Teaching in Remote/Hybrid Learning Environments (TRLE)
  • Office of State Assessment
  • Computer-Based Testing
  • Exam Schedules
  • Grades 3-8 Tests
  • Regents Exams
  • New York State Alternate Assessment (NYSAA)
  • English as a Second Language Tests
  • Test Security
  • Teaching Assistants
  • Pupil Personnel Services Staff
  • School Administrators
  • Professionals
  • Career Schools
  • Fingerprinting
  • Accountability
  • Audit Services
  • Budget Coordination
  • Chief Financial Office
  • Child Nutrition
  • Facilities Planning
  • Ed Management Services
  • Pupil Transportation Services
  • Religious and Independent School Support
  • SEDREF Query
  • Public Data
  • Data Privacy and Security
  • Information & Reporting

High School Regents Examinations

  • June 2024 Examinations General Information
  • Past Regents Examinations
  • Archive: Regents Examination Schedules
  • High School Administrator's Manual
  • January 2024 Regents Examination Scoring Information
  • June 2024 Regents Examination Scoring Information
  • Testing Materials for Duplication by Schools
  • English Language Arts
  • Next Generation Algebra I Reference Sheet
  • Life Science: Biology
  • Earth and Space Sciences
  • Science Reference Tables (1996 Learning Standards)

United States History and Government

  • Global History and Geography II
  • High School Field Testing
  • Test Guides and Samplers
  • Technical Information and Reports

General Information

  • Information Booklet for Scoring the Regents Examination in United States History and Government
  • Frequently Asked Questions on Cancellation of Regents Examination in United States History and Government  - Revised, 6/17/22
  • Cancellation of the Regents Examination in United States History and Government for June 2022
  • Educator Guide to the Regents Examination in United States History and Government - Updated, July 2023
  • Memo: January 2022 Regents Examination in United States History and Government Diploma Requirement Exemption
  • Timeline for Regents Examination in United States History and Government and Regents Examination in United States History and Government
  • Regents Examination in United States History and Government Essay Booklet - For June 2023 and beyond
  • Prototypes for Regents Examination in United States History and Government
  • Regents Examination in United States History and Government Test Design - Updated, 3/4/19
  • Performance Level Descriptors (PLDs) for United States History and Government

Part 1: Multiple-Choice Questions

  • Part I: Task Models for Stimulus Based Multiple-Choice Question

Part II: Stimulus-Based Short Essay Questions: Sample Student Papers

The links below lead to sample student papers for the Part II Stimulus-Based Short Essay Questions for both Set 1 and Set 2. They include an anchor paper and a practice paper at each score point on a 5-point rubric. These materials were created to provide further understanding of the Part II Stimulus-Based Short Essay Questions and rubrics for scoring actual student papers. Each set includes Scoring Worksheets A and B, which can be used for training in conjunction with the practice papers. The 5-point scoring rubric has been specifically designed for use with these Stimulus-Based Short Essay Questions.

Part III: Civic Literacy Essay Question

The link below leads to sample student papers for the Part III Civic Literacy Essay Question. It includes Part IIIA and Part IIIB of a new Civic Literacy Essay Question along with rubrics for both parts and an anchor paper and practice paper at each score point on a 5-point rubric. These materials were created to provide further understanding of the Part III Civic Literacy Essay Question and rubric for scoring actual student papers. Also included are Scoring Worksheets A and B, which can be used for training in conjunction with the practice papers. The 5-point scoring rubric is the same rubric used to score the Document-Based Question essay on the current United States History and Government Regents Examination.

  • Part III: Civic Literacy Essay Question Sample Student Papers

Get the Latest Updates!

Subscribe to receive news and updates from the New York State Education Department.

Popular Topics

  • Charter Schools
  • High School Equivalency Test
  • Next Generation Learning Standards
  • Professional Licenses & Certification
  • Reports & Data
  • School Climate
  • School Report Cards
  • Teacher Certification
  • Vocational Services
  • Find a school report card
  • Find assessment results
  • Find high school graduation rates
  • Find information about grants
  • Get information about learning standards
  • Get information about my teacher certification
  • Obtain vocational services
  • Serve legal papers
  • Verify a licensed professional
  • File an appeal to the Commissioner

Quick Links

  • About the New York State Education Department
  • About the University of the State of New York (USNY)
  • Business Portal for School Administrators
  • Employment Opportunities
  • FOIL (Freedom of Information Law)
  • Incorporation for Education Corporations
  • NYS Archives
  • NYS Library
  • NYSED Online Services
  • Public Broadcasting

Media Center

  • Newsletters
  • Video Gallery
  • X (Formerly Twitter)

New York State Education Building

89 Washington Avenue

Albany, NY 12234

CONTACT US  

NYSED General Information: (518) 474-3852

ACCES-VR: 1-800-222-JOBS (5627)

High School Equivalency: (518) 474-5906

New York State Archives: (518) 474-6926

New York State Library: (518) 474-5355

New York State Museum: (518) 474-5877

Office of Higher Education: (518) 486-3633

Office of the Professions: (518) 474-3817

P-12 Education: (518) 474-3862

EMAIL CONTACTS  

Adult Education & Vocational Services 

New York State Archives 

New York State Library 

New York State Museum 

Office of Higher Education 

Office of Education Policy (P-20)

© 2015 - 2024 New York State Education Department

Diversity & Access | Accessibility | Internet Privacy Policy | Disclaimer  |  Terms of Use  

IMAGES

  1. AP Comp Gov

    ap gov argument essay example 2021

  2. AP Government Argument Essay Organizer- Exam Prep Template! by Mad

    ap gov argument essay example 2021

  3. AP Argument- Score a Sample Essay

    ap gov argument essay example 2021

  4. Ap Lang Argument Essay Template

    ap gov argument essay example 2021

  5. Argument Essay Detailed Walkthrough AP Gov 2021 NEW!

    ap gov argument essay example 2021

  6. AP Gov Unit 4 FRQ.pdf

    ap gov argument essay example 2021

VIDEO

  1. 2024 AP Gov FRQ Answers!!

  2. my gov par Google se ya app se contest jaise poster ,essay, logo ya tagline me pratibhag kaise kare

  3. Essay writing competition on yoga day

  4. Ap Eamcet 2024 After Response Sheet Below 40 marks

  5. 2024 AP Gov: Entire Course 14 Minute Review!

  6. AP Gov LIVE Argument Essay Review 5/2 2 PM EST

COMMENTS

  1. PDF AP United States Government and Politics

    2021 AP Exam Administration Sample Student Responses - AP U.S. Government and Politics Free-Response Question 4: Set 1 Author: College Board Subject: 2021 AP Exam Administration: Student Samples and Commentary Keywords

  2. AP U.S. Government and Politics Exam Questions

    Note: Some questions and scoring guidelines from the 2023 and earlier AP U.S. Government and Politics Exams may not perfectly align with the course and exam updates that take effect in the 2023-24 school year. These questions remain available because teachers say that imperfectly aligned questions still provide instructional value.

  3. PDF AP United States Government and Politics

    AP® United States Government and Politics 2021 Scoring Guidelines . Question 4: Argument Essay 6 points . Reporting Category Scoring Criteria . Row A Claim/Thesis (0-1 points) 0 points . Does not meet the criteria for one point. 1 point . Responds to the prompt with a defensible claim or thesis that reasoning. Decision Rules and Scoring Notes

  4. Argument Essay Detailed Walkthrough AP Gov 2021 NEW!

    In this video I walk you through the process of writing the AP Gov Argument Essay and make sure you know exactly what you need to do your absolute best on th...

  5. AP U.S. Government and Politics: Argument Essay

    The Argument Essay differs substantially from the other free-response questions on the AP U.S. Government and Politics exam, but you can and should still follow the Kaplan Method (AP-AP). It is recommended that you take 40 minutes to plan and write your Argument Essay (as opposed to 20 minutes each for the other free-response questions), so ...

  6. AP Comp Gov

    The prompt used in this video is (mostly) from the Course and Exam Description, which you can read here: https://apcentral.collegeboard.org/pdf/ap-comparativ...

  7. AP Government: Argumentative Essay Practice

    Chose one or more of the provided Argumentative Essay Prompts, as assigned, and use the planning and exploration you did above to write a full essay in response to your designated prompt (s) in 25 ...

  8. AP Gov FRQ: Argument Essay Review (2020)

    Because of that, you should spend around 25 minutes, give or take a few, on the Argument Free-Response Question. (NOTE: FOR THE 2019-2020 TEST, YOU WILL HAVE 25 MINUTES TO WRITE AND 5 MINUTES TO UPLOAD YOUR RESPONSE.) This is the nightmare you're not gonna have before this AP exam. Image courtesy of Freepik.

  9. How to Write the ARGUMENTATIVE ESSAY FRQ for AP Gov

    More from Heimler's History:AP HEIMLER REVIEW GUIDE (formerly known as the Ultimate Review Packet): +AP Gov Heimler Review Guide: https://bit.ly/3rfXr2YCheck...

  10. The Complete Guide to AP US Government FRQs

    The free-response section lasts one hour and 40 minutes and consists of four questions, each of which is worth 12.5% of your total score. So as a whole, the free-response section accounts for half your total AP Gov score (the other 50% comes from the multiple-choice section). Each FRQ is worth 3-6 raw points.

  11. AP U.S. Government and Politics Exam Tips

    AP U.S. Government and Politics Exam Tips. The following strategies for answering the free-response questions will help you on exam day. Answering essay questions generally requires a good deal of training and practice. Students too often begin to write immediately, which can create a string of disconnected, poorly planned thoughts.

  12. PDF AP United States Government and Politics

    This Argument Essay question expected students to demonstrate an understanding of how outside groups influence federal policy makers to effect changes in policy in the area of civil rights, as well as an understanding of foundational documents related to the concept of competing policy making

  13. Sample Prompts for the Argument Essay FRQ- AP government

    Sample Prompts for the Argument Essay FRQ- AP government. Below are 16 topics, each of which includes: A sample essential question which introduces the prompt. A draft prompt including three founding documents that could help shape the students' arguments. Each prompt is crafted to encourage deep analysis and aligns with key AP Government ...

  14. How to Answer AP® US Government Free Response Questions

    1. Commit to learning what gets you points on the AP® US Government and Politics exam by reviewing past rubrics and scoring guidelines. 2. Underline or circle every bolded and capitalized word in the question prompt. 3. Plan your response BEFORE beginning to write your response. 4.

  15. The Complete Guide to the AP Comparative Government and Politics Exam

    Sample Question: Argument Essay Free-Response. ... Take a look at the table below to see what percentage of test takers earned each possible scaled score on the 2021 AP Comparative Government exam: AP Score % of Students Who Earned Score (2021) 5. 16.6. 4. 24.5. 3. 30.7. 2. 14.9. 1. 13.3.

  16. AP U.S. Government and Politics: Concept Application Example

    AP U.S. Government and Politics: Concept Application Example. Question 1 of the AP U.S. Government and Politics free-response section will always be the Concept Application prompt. This prompt will begin with a stimulus that is a short paragraph or two describing a political scenario. The paragraph (s) could be an excerpt from news media, a ...

  17. How to Write the Argument Essay AP Gov

    Everything you need to know about how to write the Argument Essay! Check out the AP Gov Ultimate Review Packet: https://www.ultimatereviewpacket.com/courses/...

  18. How to Write the AP Lang Argument Essay (With Example)

    The sample prompt below is published online by the College Board and is a real example from the 2021 AP English Exam. The prompt provides background context, essay instructions, and the text you need to analyze. ... AP Lang Argument Essay Example: Answer Breakdown. The sample AP Lang Argumentative Essay above has some strengths and some ...

  19. PDF AP Comparative Government and Politics

    Government and Politics Sample Student Responses and Scoring Commentary Set 2 Inside: ... AP® Comparative Government and Politics 2021 Scoring Guidelines ... Argument Essay 5 points . Reporting Category . Scoring Criteria ; Row A ; Claim/Thesis (0-1 points) 0 points . Does not meet the criteria for one point.

  20. Argumentative Essay Examples 1.pdf

    View Argumentative_Essay_Examples_1.pdf from LANGUAGE 101 at High School Of Economics & Finance. 2021 AP Comparative Government and Politics ® Sample Student Responses and Scoring Commentary Set

  21. How to Write the AP Lang Argument Essay + Examples

    2. Pick one side of the argument, but acknowledge the other side. When you write the essay, it's best if you pick one side of the debate and stick with it for the entire essay. All your evidence should be in support of that one side. However, in your introductory paragraph, as you introduce the debate, be sure to mention any merit the ...

  22. APGOV: Argumentative Essay Questions : r/APStudents

    APGOV: Argumentative Essay Questions. We literally covered this in class in, like, the last two days so I have questions before the exam tomorrow. May post more questions about this in the comments, but here's one with an example: Say I have three documents to work with: The Declaration of Independence, Fed 51, and Fed 70.

  23. AP United States Government and Politics Exam

    Section I: Multiple Choice. 55 Questions | 1 Hour 20 Minutes | 50% of Exam Score. Individual questions (no stimulus): ~30. Set-based questions. Quantitative Analysis: Analysis and application of quantitative-based source material. Qualitative Analysis: Analysis and application of text-based (primary and secondary) sources.

  24. United States History and Government

    Part II: Stimulus-Based Short Essay Questions: Sample Student Papers. The links below lead to sample student papers for the Part II Stimulus-Based Short Essay Questions for both Set 1 and Set 2. They include an anchor paper and a practice paper at each score point on a 5-point rubric.